How many pounds of each type of fruit did she buy? she bought
pounds of limes and pounds of pears.

Answers

Answer 1

Pat bought 5 pounds of apples.

(1) 1 pound of pears cost $0.5 more that 1 pound of apples.

If 1 pound of pears cost $1 and 1 pound of apples cost $0.5, then the cost of 5 pounds of apples is 5*0.5=$2.5. For $2.5 we can buy 2.5/1=2.5 pounds of pears.

If 1 pound of pears cost $1.5 and 1 pound of apples cost $1, then the cost of 5 pounds of apples is 5*1=$5. For $5 we can buy 5/1.5=10/3 pounds of pears.

(2) 1 pound of pears cost 1.5 times as much as 1 pound of apples.

The cost of 5 pounds of apples is $5a (where a is the cost of 1 pound of apples). For $5a we can buy 5a/(1.5a)=5/1.5 pounds of pears. Sufficient.

Learn more about  total cost at:

https://brainly.com/question/25109150

#SPJ1

Given question is incomplete , the complete question is given below ,

Pat bought 5 pounds of apples. How many pounds of pears could he have bought for same amount of money?

(1) 1 pound of pears cost $0.5 more that 1 pound of apples

(2) 1 pound of pears cost 1.5 times as much as 1 pound of apples


Related Questions

A ream of a certain brand of paper weighs about 4.533 pounds. A ream contains 500 sheets of paper. How much does a sheet of paper​ weigh?

Answers

Step-by-step explanation:

As a ream or

500

sheets of paper weigh

4.818

pounds

One sheet of paper weighs

4.818

500

=

0.009636

pounds.

It is apparent that pound is too big a unit for a sheet of paper.

As each pound has

16

ounces, one can say

one sheet of paper weighs

0.009636

×

16

=

0.154176

ounces.

If ounce is too big, as we have

1

pound equal to

28.34952

grams

one sheet of paper weighs

0.154176

×

28.34952

4.371

grams

please mark as brainliest


EX2. Find the solution of the following matrix using
Gauss elimination method, working 4D.
2.37 X1+3.06 X-4.28X, -1.76
1.46 Xi - 0.78 X+3.75X4.69
-3.69 X: +5.13 X:-1.06X, 5.74

Answers

Answer:

please post a more cohesive question. The values are very confusing

HELP MATH ANSWER QUESTION

Answers

Answer:

x = 84 , y = 168

Step-by-step explanation:

[tex]Sin 60 = \frac{opposite side}{hypotenuse}\\\\ \frac{\sqrt{3}}{2}= \frac{AB}{BC}\\\\ \frac{\sqrt{3}}{2} = \frac{84\sqrt{3}}{y}\\\\Cross\ multiply ,\\\\y*\sqrt{3} =2*84 \sqrt{3}\\\\y= \frac{2*84*\sqrt{3}}{\sqrt{3}}\\\\y=168[/tex]

[tex]Cos\60 = \frac{adjacent}{hypotenuse}\\\\ \frac{1}{2}= \frac{AC}{BC}\\\\ \frac{1}{2}= \frac{x}{168}\\\\ \frac{1}{2}*168=x\\\\x= 84[/tex]

The difference between two numbers is 7194. If the subtrahend is 12,806, find the minuend.

Answers

Answer: The minuend is 20000

Step-by-step explanation:

To calculate the minuend, we use the equation:

[tex]\text{Subtrahend}=\text{Minuend-Difference}[/tex]

We are given:

Difference between the two numbers = 7194

Subtraend = 12806

Putting values in above equation, we get:

[tex]\text{Minuend}=12806+7194\\\\\text{Minuend}=20000[/tex]

Hence, the minuend is 20000

How many distinct 5 digit numbers can be formed from the list of numbers
{0,1,2,3,4,5,6,7,8,9}such that the number
i. Is an even number
ii. Is a Odd number
iii. Multiple of 5
iv. Never ends with {2,3,4,5}

Answers

Case 1: A 5 digit number with the digits {1,2,3,4,5}. Since 0 is not a part of this set and there are 5 different digits, we can form a total of 5P5 = 5! = 120 numbers.

How the formula for compound interest is derived?please explain in easy way

Answers

Answer - Compound interest is calculated by multiplying the initial principal amount by one plus the annual interest rate raised to the number of compound periods minus one. The total initial amount of the loan is then subtracted from the resulting value.

Solve this pls help


How did the Californian gold rush have an effect on people?

Answers

Answer:

The gold rush has a big impact on people. Especially since it helped shaped the course of California's development by bringing economic growth.

The price of a pen is Rs 5 more than the price of a pencil. Total price of 3 pencil and 2 pen is Rs 45.

a) Take the price of pencil as x write the price of pen in terms of x ?

b) Find the price of pencil and pen by forming an equation ?

Answers

Answer:

See below

Step-by-step explanation:

a) y = 5x

b) 45 = 3y + 2x

Help ASAP
What is the solution to this equation? (1/4)^x+1 =32

Answers

Answer:

B.  [tex]\displaystyle \frac{-7}{2}[/tex]

General Formulas and Concepts:

Pre-Algebra

Order of Operations: BPEMDAS

Brackets Parenthesis Exponents Multiplication Division Addition Subtraction Left to Right  

Equality Properties

Multiplication Property of Equality Division Property of Equality Addition Property of Equality Subtraction Property of Equality

Algebra I

Exponential Rule [Powering]:                                                                           [tex]\displaystyle (b^m)^n = b^{m \cdot n}[/tex] Exponential Rule [Rewrite]:                                                                              [tex]\displaystyle b^{-m} = \frac{1}{b^m}[/tex]

Step-by-step explanation:

Step 1: Define

Identify

[tex]\displaystyle \bigg( \frac{1}{4} \bigg)^{x + 1} = 32[/tex]

Step 2: Solve for x

Rewrite:                                                                                                             [tex]\displaystyle \bigg( \frac{1}{2^2} \bigg)^{x + 1} = 2^5[/tex]Exponential Rule [Rewrite]:                                                                              [tex]\displaystyle (2^{-2})^{x + 1} = 2^5[/tex]Exponential Rule [Powering]:                                                                          [tex]\displaystyle 2^{-2(x + 1)} = 2^5[/tex]Set:                                                                                                                    [tex]\displaystyle -2(x + 1)} = 5[/tex][Division Property of Equality] Divide -2 on both sides:                                [tex]\displaystyle x + 1 = \frac{-5}{2}[/tex][Subtraction Property of Equality] Subtract -1 on both sides:                        [tex]\displaystyle x = \frac{-7}{2}[/tex]

At the start of 2012, the US federal budget had a deficit of more than $1.5×1013. Convert this number to decimal form.

Answers

The decimal form of the given number is $15,000,000,000,000.00

Conversion to decimal

From the question, we are to convert the given number to decimal form

The given number is $1.5×10¹³

$1.5×10¹³ = $15,000,000,000,000.00

Hence, the decimal form of the given number is $15,000,000,000,000.00

Learn more on Conversion to decimals here: https://brainly.com/question/10944785

#SPJ1

Plz help. I was crying. Super challenging. Will give brainiest and will give 50 points for explained answer. Thx

Answers

Answer:

Paige is incorrect.  g(x) has a steeper slope

Step-by-step explanation:

The slope for f(x)  is

m =( y2-y1)/(x2-x1)

   = (17/2 - 5/2) / ( -1 - -6)

    = (17/2 - 5/2) / ( -1 +6)

    = (12/2)/5

    = 6/5

The slope for g(x)  is

m =( y2-y1)/(x2-x1)

   = (-1 - 13/3) / ( 6 - 2)

    =(-3/3 -13/3) / (6-2)

    (-16/3)/4

     -16/3 * 1/4

   - 4/3

Comparing the magnitudes

|6/5|   |-4/3|

|6/5*3/3|     |-4/3*5/5|

|18/15|           |20/15|

|20/15| is greater so it has a steeper slope

g(x) has a steeper slope

60.3 60.44 witch one is greater

Answers

60.44 is greater than 60.3
-you can see this when you subtract 60.3 from 60.44 this would leave a 0.11 which is a positive number.
-thus meaning 60.44 is larger/greater.

Question
Which of the following is equivalent to 1-12 + 3/?

Answers

d because if you look and add them there the exact the same

Which expression is equivalent to the given expression?

6ab/(a^0b^2)^4

Answers

Answer:

,here is the answer

Step-by-step explanation:

here is your answer

I took 18 pencils from the box. this is equivalent to 2/5 of the total number of pencils.
How many pencils were there in the box originally. Show your work.

Answers

45
if 18 is 2/5, than 9 is 1/5. That means that 5/5 is 45.
Hope this helps!

Answer: 45 pencils were in the box originally

Step-by-step explanation:

Suppose there were x pencils in the box originally. Since 18 pencils is equal to 2/5 of the total number of pencils in the box originally, set 18 equal to 2/5x or

18=2/5x

Now solve

x = 18x5/2 = 45

Suppose we are sending a digital signal which is a string of 0s and 1s of length five. (Example stringsare 00101, 11000, 10101 are all 5 bit strings.) When we send the message, each bit (0 or 1) is sentindependently and there is some chance that the bit is corrupted. Namely, each time we send a 0 thereis a 5% chance that a 1 is received and each time we send a 1 there is a 5% chance a 0 is received.Suppose we send a message of length 5, what is the probability that an incorrect message is received

Answers

Answer:

0.7738 = 77.38% probability that an incorrect message is received.

Step-by-step explanation:

For each bit, there are only two possible outcomes. Either it is corrupted, or it is not. The probability of a bit being corrupted is independent of any other bit. This means that the binomial probability distribution is used to solve this question.

Binomial probability distribution

The binomial probability is the probability of exactly x successes on n repeated trials, and X can only have two outcomes.

[tex]P(X = x) = C_{n,x}.p^{x}.(1-p)^{n-x}[/tex]

In which [tex]C_{n,x}[/tex] is the number of different combinations of x objects from a set of n elements, given by the following formula.

[tex]C_{n,x} = \frac{n!}{x!(n-x)!}[/tex]

And p is the probability of X happening.

5% probability a bit is sent incorrectly:

This means that [tex]p = 0.05[/tex]

Message of length 5

This means that [tex]n = 5[/tex]

What is the probability that an incorrect message is received?

This is the probability of at least one incorrect bit, which is:

[tex]P(X \geq 1) = 1 - P(X = 0)[/tex]

In which

[tex]P(X = x) = C_{n,x}.p^{x}.(1-p)^{n-x}[/tex]

[tex]P(X = 0) = C_{5,0}.(0.05)^{0}.(0.95)^{5} = 0.7738[/tex]

0.7738 = 77.38% probability that an incorrect message is received.

Assume that you purchased a new car today and financed $55,000 of the price on a 72-month payment contract with a nominal rate of 6.00%. Further, assume that you plan on paying off the balance of the car loan after you make your 48th payment. How much will your loan balance be when you pay off the car?

Answers

Answer:

The amount that your loan balance will be when you pay off the car is $20,566.18.

Step-by-step explanation:

Step 1. Calculation of monthly payment

This can be calculated using the formula for calculating the present value of an ordinary annuity as follows:

PV = P * ((1 - (1 / (1 + r))^n) / r) …………………………………. (1)

Where;

PV = Present value or the cost of the new car = $55,000

P = Monthly payment = ?

r = Monthly nominal rate = Nominal rate / 12 = 6% / 12 = 0.06 / 12 = 0.005

n = number of months = 72

Substitute the values into equation (1) and solve for P, we have:

$55,000 = P * ((1 - (1 / (1 + 0.005))^72) / 0.005)

$55,000 = P * 60.3395139355201

P = $55,000 / 60.3395139355201 = $911.51

Step 2. Calculation of the loan amount balance when you pay off the car

This can be calculated using the ballon payment formula as follows:

P = (PV - (Ballon / (1 + r)^n)) * (r / (1 – (1 + r)^-n)) ...................... (1)

Where:

P = Monthly payment = $911.51

PV = Present value or the cost of the new car = $55,000

Ballon = Ballon payment or the loan amount balance when you pay off the car = ?

r = Monthly nominal rate = Nominal rate / 12 = 6% / 12 = 0.06 / 12 = 0.005

n = number months to pay off the loan amount balance = 48

Substituting the values into equation (1) and solve for Ballon, we have:

911.51 = (55,000 - (Ballon / (1 + 0.005)^48)) * (0.005 / (1 - (1 + 0.005)^-48))

911.51 = (55,000 - (Ballon / 1.27048916109538)) * 0.0234850290479363

911.51 / 0.0234850290479363 = 55,000 - (Ballon / 1.27048916109538)

38,812.39 = 55,000 - (Ballon / 1.27048916109538)

Ballon / 1.27048916109538 = 55,000 - 38,812.39

Ballon / 1.27048916109538 = 16,187.61

Ballon = 16,187.61 * 1.27048916109538

Ballon = $20,566.18

Therefore, the amount that your loan balance will be when you pay off the car is $20,566.18.

find the surface area of the composite figure​

Answers

Answer:

[tex]=280[/tex] [tex]in^2[/tex]

Step-by-step explanation:

----------------------------------------

Let's find the surface area of the pink rectangular prism first.

[tex]2*10=20+20=40[/tex]

[tex]4*10=40+40=80[/tex]

[tex]4*2=8+8=16[/tex]

[tex]40+80+16=136[/tex]

The surface area for the pink rectangular prism is [tex]136[/tex] [tex]in^2[/tex].

-------------------->>>>>

Now, let's find the surface area of the green rectangular prism.

[tex]4*7=28+28=56[/tex]

[tex]4*7=28+28=56[/tex]

[tex]4*4=16+16=32[/tex]

[tex]56+56+32=144[/tex]

The surface area for the green rectangular prism is 144 [tex]in^2[/tex].

-------------------->>>>>

Now let's add the surface area of both rectangular prisms to find the surface area of the composite figure.

[tex]136+144=[/tex]

[tex]=280[/tex] [tex]in^2[/tex]

----------------------------------------

Hope this is helpful.

9514 1404 393

Answer:

  224 in²

Step-by-step explanation:

There are a couple of ways to go at this. Here, we choose to figure the areas of each of the prisms individually, then subtract the "hidden" area where they are joined together.

The area of a prism is ...

  A = 2(LW +H(L+W))

Pink area:

  A = 2(10·4 +2(10+4)) = 2(40 +28) = 136 . . . square inches

Green area:

  A = 2(7·4 +4(7+4)) = 2(28 +44) = 144 . . . square inches

One 4 in × 7 in face of the green prism meets with a similar area of the pink prism, so the area hidden at that interface is 2(4·7) = 56 square inches. Then the total surface area of the composite figure is ...

  SA = 136 in² +144 in² -56 in² = 224 in²

Which polynomial is factored completely?

Answers

Answer:

You answered it

image. can someone help with this one uWu​

Answers

Answer:

8/33

Step-by-step explanation:

Three potential employees took an aptitude test. Each person took a different version of the test. The scores are reported below. Kerri got a score of 80.8; this version has a mean of 62.1 and a standard deviation of 11. Cade got a score of 286.4; this version has a mean of 271 and a standard deviation of 22. Vincent got a score of 7.9; this version has a mean of 7.2 and a standard deviation of 0.7. If the company has only one position to fill and prefers to fill it with the applicant who performed best on the aptitude test, which of the applicants should be offered the job

Answers

Answer:

Kerri

calculate z scores z= (x - xbar)/stdev

Kerri = 1.7

Cade = .7

Vincent = 1

Step-by-step explanation:

What is the probability of flipping a coin 10 times and getting heads 5 times? Round your answer to the nearest tenth percent.

Answers

The Probability is 1/2 or .5, .5 rounded to the nearest tenth would be irregular because .5 is already in the tenth place but according to the law of rounding since ut is five it would be rounded up giving the whole number 1

Show that the transformation T defined by ​T(x1​, x2​)= ​(x1x2​, x1​, x2​) is not linear. If T is a linear​ transformation, then ​T(0​)= ____________

Answers

Answer:

A linear transformation is defined as:

For a transformation T that goes from R^n to R^m, this transformation is linear if, for two vectors A and B, we have that:

T(A + B) = T(A) + T(B)

In this case, we have the transformation:

T(x1​, x2​)= ​(x1x2​, x1​, x2​).

If we define two vectors:

A = (a1, a2)

B = (b1, b2)

if the transformation is lineal, we will have that:

T(A + B) = T(A) + T(B)

Or

T(0) = T(A - A) = T(A) + T(-A)

(just two different ways of writing the same thing, I will use the first one, because it is the general way)

We want to see that, for our transformation, this equation is false.

first the left side:

A + B = (a1, a2) +  (b1, b2) = (a1 + b1, a2 + b2)

Then the transformation applied to that vector gives:

T(A + B) = T(a1 + b1, a2 + b2) = ( (a1 + b1)*(a2 + b2), (a1 + b1), (a2 + b2))

             = (a1*a2 + a1*b2 + b1*a2 + b1*b2, a1 + b1, a2 + b2)                                  

While for the right side, we have:

T(A) + T(B) = T(a1, a2) + T(b1, b2) =  (a1*a2, a1, a2) + (b1*b2, b1, b2)

                  = (a1*a2 + b1*b2, a1 + b1, a2 + b2)

Then we can rewrite:

T(A + B) = T(A) + T(B)

as:

(a1*a2 + a1*b2 + b1*a2 + b1*b2, a1 + b1, a2 + b2) = (a1*a2 + b1*b2, a1 + b1, a2 + b2)

We can see that the first part of these vectors is different, thus, the equality is false.

Then we can conclude that:

T(A + B)  ≠ T(A) + T(B)

Then the transformation T is not linear.

Find(g-f)(x)
f(x)=1/x
g(x)=x^2+2x-5

a. 1/x-x^2-2x+5
b.1/x+x^2+2x-5
c.x^2+2x-5-1/x
d.1/x+x^2-5

Answers

Answer:

x^2+2x-5 - 1/x

Step-by-step explanation:

f(x)=1/x

g(x)=x^2+2x-5

(g-f)(x) = x^2+2x-5 - 1/x

Answer:

c. x² + 2x - 5 - 1 / x

Step-by-step explanation:

f ( x) = 1 / x.

g ( x) = x² + 2x - 5

so,

( g - f) ( x) = x² + 2x - 5 - 1 / x

A. If I do something for 30 seconds every day for a whole year, how long did I do it in total.

B. If I do something for 10 seconds every day for a whole year, how long did I do it in total.​

Answers

Answer: 120 minutes

Step-by-step explanation:

Answer:

A. 10,950 seconds, 182.50 minuets, and about 3 hours

B. 3,650 seconds, about 60 minuets, or about 1 hour.

Step-by-step explanation:

A. We can multiply the seconds by the days in a year to get our answer, and we can simply divide if we want to get the amount of minuets or hours.

B. Same thing as "A", but all we do here, is multiply 10 by 365. :)

Which option shows the graph of 3y+18>5x

Answers

Hi, I'm so sorry I can't answer that. but I have something you can answer that. Search in browser or anything searcher that you have "Quickmath" or "Cymath" all your equations,problems can solved

Step-by-step explanation:

i hope it helps

What is this function’s input if its output is 11?

f(x) = 2x + 5

Answers

Answer:

the input x is 3

Step-by-step explanation:

2x+5=11

   2x=6

     x=3

the input is 3 #speakingfromthehesrtbfletcher

what is the best estimate of (-3/8)(17 5/6)​

Answers

Answer:

-11

Step-by-step explanation:

Given the expression (-3/5)(17 5/6)

Converting the mixed to improper fraction

​(-3/5)(17 5/6)

=  (-3/5)(107/6)

= (-3*107)/(5*6)

= -321/30

= -10.7

Hence the required best estimate is -11

There are 12 adults, 24 girls and 6 boys in a room.
Work out the ratio of adults to girls to boys.
Write the answer in its simplest form.

Answers

Answer:

12:24:6- divide by 3

4:8:2 - divide by 2

2:4:1- simplest form

The initial number of views for a reader board is 25. The number of views is growing exponentially at a rate of 18% per week. What is the number of views expected to be four weeks from now?

Answers

Answer:

48

Step-by-step explanation:

n = 25(1.18)^4

n = 48.469444

Rounded to 48

Answer:

48 views.

Step-by-step explanation:

18% = 0.18 as a decimal fraction.

The increase in number of books per week is  found by multiplying by 1.18.

The equation is  an exponential one and is:

V = 25(1.18)^t  where V = views and t = number of weeks.

So after 4 weeks :

V = 25 (1.18)^4

= 48.47

Other Questions
A test bar of nonferrous material has a diameter of 0.253 inches. Upon applying a tensile load, the sample exhibited 0.002 plastic strain at 3400 lb and the maximum load during testing was 6200 lb and occurred at an engineering strain of 0.65; and breaking occurred at 4400 lb. The sample diameter at fracture was measured to be 0.15 inches. Required:a. The yield strength of the material is :________b. The UTS of the material is:________ Prepare journal entries to record each of the following transactions. The company records purchases using the gross method and a perpetual inventory system. Sept. 15 Purchased merchandise with an invoice price of $77,500 and credit terms of 3/5, n/15.Sept. 29 Paid supplier the amount owed on the September 15 purchase. The owner of Artisanal Chips etc. produces three flavors of artisanal corn chips marketed at new college graduates pumpkin (P), chipotle adobo (A) and basement (B). He has a limited amount of the three ingredients used to produce these chips available for his next production run: 1,000 ounces of salt, 2,000 ounces of maize, and 1,200 ounces of herbs. A bag of pumpkin chips requires 2 ounces of salt, 6 ounces of maize, and 1.75 ounces of herbs to produce; while a bag of chipotle-adobo chips requires 6 ounces of salt, 6 ounces of maize, and 5 ounces of herbs. A bag of basement chips requires 1.75 ounces of salt, 3.5 ounces of maize, and 1.5 ounces of herbs. Profits for a bag of basement chips are $0.40, for chipotle-adobo chips is $0.60, and for a bag of pumpkin chips $0.50.For the production combination of 100 bags of each flavor of chips, which of the three resources is (are) not completely used? a. salt and herbs only b. salt and maize only c. herbs maize and salt d. maize only The diagram below represents a beaker of water being heated by a flame. The arrows represent heattransfer occurring in the beaker.Which process is represented by the arrows in the diagram?O condensationO conductionO convectionO radiation trnh by ngn gn hiu bit ca bn thn v mng my tnh theo m hnh OSI hoc TCP/IP How bullet train and a circuit breaker work on the magnet effect of current Think Critically 5-1 Creating Flexible Storage. It's been 6 months since the disk crash at CSM Tech Publishing, and the owner is breathing a little easier because you installed a fault-tolerant solution to prevent loss of time and data if a disk crashes in the future. Business is good, and the current solution is starting to get low on disk space. In addition, the owner has some other needs that might require more disk space, and he wants to keep the data on separate volumes. He wants a flexible solution in which drives and volumes aren't restricted in their configuration. He also wants to be able to add storage space to existing volumes easily without having to reconfigure existing drives. He has the budget to add a storage enclosure system that can contain up to 10 HDDs. Which Windows feature can accommodate these needs, and how does it work? Compound A has a partition coefficient (K) of 7 when comparing its solubility in CH2Cl2 to water ( K=11, [solubility of A in g/ml in CH2Cl2] divided by [solubility of A in g/ml in H2O]). If we take 17.0 g of A and partition it thoroughly between 150 ml of CH2Cl2 and 100 ml of water, what is the equation which will tell us how much of A (which is represented by x) is in the water layer after this partitioning ? Find the measure of WX and show work The forgery ________ so exquisite that it hung in the museum for the past two decades without discovery.Which verb or verb phrase is in the correct tense to complete the sentence? What was true of the Harappan civilization?A. They never created a writing system.B. They never built permanent homes.C. They practiced Hinduism.D. They traded with their neighbors. find the measure of the missing angle (#4 btw) Doris has a fish pond 15 by 18 ft. She wants to put grass in a strip of uniform with around the pond. She has enough grass seed for 234 ft squared. How wide will the strip be? Round your answer to the nearest tenth, when appropriate Consider rolling two fair dice and observing the number of spots on the resulting upward face of each one. Letting A be the event that at least one of the dice results in 6 spots on its upward face, and E be the event that exactly one of the dice results in 6 spots on its upward face, give the value of P(E|A). (Note: This value can be easily obtained by looking at p. 2.3 of the class notes and using a reduced sample space. In fact, such an approach makes the solution so trivial, that for this problem, you dont have to give any justification for your answer.) the mean age of 8 people is 16 years. when mrs. hernandez's age is included, the mean age increases to 20. how old is mrs. hernandez? M^3 is a derived unit why? Consider the case of the following annuities, and the need to compute either their expected rate of return or duration. Joshua inherited an annuity worth $6,830.77 from his uncle. The annuity will pay him eight equal payments of $1,100 at the end of each year. The annuity fund is offering a return __________Joshua's friend, Willie, has hired a financial planner for advice on retirement. Considering Willie's current expenses and expected future lifestyle changes, the financial planner has stated that once Willie crosses a threshold of $1,387,311 in savings, he will have enough money for retirement. Willie has nothing saved for his retirement yet, so he plans to start depositing $25,000 in a retirement fund at a fixed rate of 6.00% at the end of each year. It Will take ____________ for Willie to reach his retirement goal. ??? Whats the answer Seven times a number decreased by three-fourths of the same number is 25. What is the number? For each of the following scenarios, determine whether the decision maker is risk neutral, risk averse, or risk loving. a. A manager prefers a 20 percent chance of receiving $1,400 and an 80 percent chance of receiving $500 to receiving $680 for sure.b. A shareholder prefers receiving $920 with certainty to an 80 percent chance of receiving $1,100 and a 20 percent chance of receiving $200.c. A consumer is indifferent between receiving $1,360 for sure and a lottery that pays $2,000 with a 60 percent probability and $400 with a 40 percent probability.